Measure Zero Sets: Proving \sigma(E) Has Measure Zero

Click For Summary
SUMMARY

The discussion centers on the proof that if a set E has measure zero, then the set σ(E) defined as σ(E) = {(x,y): x-y ∈ E} also has measure zero. The participant attempts to demonstrate that if σ(E) is not of measure zero, then there exists a point in E such that σ({e}) has positive measure. They have established that if E is open or a G_δ set, then σ(E) is measurable, which is a crucial step in the proof process.

PREREQUISITES
  • Understanding of measure theory concepts, specifically measure zero sets.
  • Familiarity with the properties of measurable sets in R^2.
  • Knowledge of G_δ sets and their implications in measure theory.
  • Basic understanding of functions and mappings in real analysis.
NEXT STEPS
  • Study the properties of measure zero sets in detail.
  • Learn about G_δ sets and their role in measure theory.
  • Explore the implications of measurable functions in R^2.
  • Investigate counterexamples related to measure zero and positive measure sets.
USEFUL FOR

Mathematicians, students in real analysis, and anyone studying measure theory who seeks to understand the properties of measure zero sets and their implications in higher dimensions.

Kindayr
Messages
159
Reaction score
0

Homework Statement


Let \sigma (E)=\{(x,y):x-y\in E\} for any E\subseteq\mathbb{R}. If E has measure zero, then \sigma (E) has measure zero.

The Attempt at a Solution


I'm trying to show that if \sigma (E) is not of measure zero, then there exists a point in E such that \sigma (\{e\}) that has positive measure. But i don't know if this actually proves the question.

I have already shown that if E open or a G_{\delta} set, then \sigma (E) is also measurable. Can I use these to solve this?

Any help is appreciated.
 
Physics news on Phys.org
The set you've defined, \sigma (E) is a subset of R^2. \sigma (\{e\}) has zero measure. It's a line in R^2.
 
Question: A clock's minute hand has length 4 and its hour hand has length 3. What is the distance between the tips at the moment when it is increasing most rapidly?(Putnam Exam Question) Answer: Making assumption that both the hands moves at constant angular velocities, the answer is ## \sqrt{7} .## But don't you think this assumption is somewhat doubtful and wrong?

Similar threads

  • · Replies 2 ·
Replies
2
Views
2K
Replies
2
Views
2K
Replies
6
Views
2K
  • · Replies 4 ·
Replies
4
Views
2K
Replies
2
Views
1K
  • · Replies 1 ·
Replies
1
Views
2K
  • · Replies 2 ·
Replies
2
Views
1K
  • · Replies 2 ·
Replies
2
Views
2K
  • · Replies 1 ·
Replies
1
Views
2K
  • · Replies 3 ·
Replies
3
Views
4K